LSAT and Law School Admissions Forum

Get expert LSAT preparation and law school admissions advice from PowerScore Test Preparation.

 Administrator
PowerScore Staff
  • PowerScore Staff
  • Posts: 8916
  • Joined: Feb 02, 2011
|
#40319
Complete Question Explanation
(The complete setup for this game can be found here: lsat/viewtopic.php?t=6748)

The correct answer choice is (D)

This Cannot Be True question tests the main inference in the game, namely, that W cannot be the first report in the first segment. If you had this inference handy, answer choice (D) would have been a no-brainer. However, if you are like most test-takers who solved this game, you did not have the requisite Not-Law on the first position. This would be unnerving given the prospect of trying out each and every answer choice. There is (almost always) an easier way to proceed.

Whenever you stumble upon a Global, Cannot Be True question that cannot be answered by referring to your initial set of Not Laws, return to the List question and see what could be true. Here, the correct answer to Question #1 was answer choice (B), in which N is the first report in the first segment. Therefore, we can safely eliminate answer choice (B) in Question #4. Next, examine the local diagrams already made. For instance, our solution for Question #2 shows that N could be the first report in either segment, helping us eliminate answer choice (C) in Question #4.

We are now left with answer choices (A), (D), and (E). One approach would be to test each of these three possibilities: two will work, and one will not (this will be the correct answer choice). A superior strategy, however, would be to skip this question entirely (for now) and tackle a few more local questions in the hopes of creating additional local diagrams. And indeed, the local solutions to Question #6 show that W can be the last report in the second segment, further eliminating answer choice (E).

With only two contenders left, creating the local diagrams will not be terribly time-consuming. Of course, you would not need to do any of this if you had examined the implications of the last rule more thoroughly, and concluded that W cannot be the first report in the first segment.

Get the most out of your LSAT Prep Plus subscription.

Analyze and track your performance with our Testing and Analytics Package.